You are on page 1of 27

INTERNATIONAL COLLEGE OF FINANCIAL PLANNING

WORKBOOK 1. Mr. Nirmal is a 30 year old, self employed person using PPF to accumulate Rs. 30000 per year. He has been saving for the last 5 years. He is willing to look at a lifestyle after retirement that fits into a fixed Rs. 3lakhs p.a. spend for 15 years. What is the spending opportunity for Nirmal, at the time of his retirement of age 60 given his savings and assume a rate of 6% on his funds after retirement. (Compounded annually, assume beginning). a. 14,26,890 b. 20,81,345 c. 24,94570 d. 18,54,425 Solution C Pmt = -30000, n=35, I =8%, mode = begin, fv = 5583064.439 Pmt = 300000, n =15, I =6%, mode = begin, PV = 3088495.176 5583064.439 3088495.176 = 2494569 2. Rahul retired from PTC Ltd. After a service of 29 years 9months. Salary at retirement was Rs.10500 pm, while average salary drawn for the preceding 10 months was Rs. 9800pm. The actual amount of gratuity received by him at the time of retirement was Rs 325000/-. Calculate the amount of gratuity exempt from tax, assuming that Rahul is covered by payment of Gratuity Act 1972? a. 315000 b. 175673 c. 181730 d. 294000 Solution C As he is covered under the gratuity act the amount of gratuity as calculated by the act will be exempt subject to a max of 350000/-. 15/26 X 10500 X 30 = 181730.769 3. Ms. Mamta is 30 and plans to retire at 58 years. Her CFP says that Mamta will require inflation adjusted Rs.75000 in the first month after retirement. Inflation is 4%p.a. & return on investment is 6%p.a. Whatll be the corpus at the time of retirement in order to meet this? Will Mamtas corpus be enough to fund her retirement if she saves Rs. 2lakhs pa. ?(end of year) . Life expectancy 75 years. a. Yes, Rs. 1,30,41,852 b. Yes, Rs. 17,48,948 c. Yes, Rs. 19,45,782 d. No, Rs. 15,78,498 Solution A Inflation adjusted rate = ((1.06/ 1.04)-1/) x 100 = 1.923076 /12 = 0.160256410

Mode = end, n = 17 x 12 = 204, I = 0.160256410, pmt = 75000, PV = 13041852.21 N = 28, mode = end, pmt = -200000, I = 6%, FV = 13705622.32 4. A perpetual bond of Rs.1000 is selling at Rs. 930. The coupon rate is 14.5% and the discounted rate is 15%. Whatll be the value of this bond and YTM? a. Rs. 950 & 15% b. Rs. 966.67 & 15.59% c. Rs950 & 14.42% d. Rs.900 & 13.5% Solution B Value of a perpetual bond = pmt / I = 145/15% = 966.67; YTM = 145/930 = 15.59% 5. Mr. Ashay 35 years wants to retire at 60; he has a life expectancy of 75 years. Current expenses are Rs.300000 annually. He estimates no reduction of expenses post retirement. How much should he save per annum to achieve his target, if inflation is 6% and yield from investment is 10%. He does not wish to leave an estate. a. Rs 153475 b. Rs.143789 c. Rs 128954 d. None Solution A = 153475 Fv of expenses; n = 25, mode end, I = 6%, PV = -300000; FV = 1287561.216 Mode = begin, n = 15, I = 3.773584%, pmt = 1287561.216, PV = -15093801.73 Mode = end, n = 25, I = 10%, FV = 15093801.73, pmt = 153474.8656 6. Mr. Prasad is working with XYZ as an accountant for the last 20 years. To supplement his earnings he was doing part time assignment in partnership entity ABC associates. He retired from both at 60 years and received Rs. 3lakh and Rs. 1 lakh as gratuity from XYZ & ABC. In the previous year. What is the maximum amount of gratuity received by him which is exempt from tax? Both the organizations are covered under the payment of gratuity act. a. Rs. 3lakhs b. Rs. 3.5 lakhs c. Rs.1lakh d. Entire 4 lakh is exempt since it is received from 2 employers. Solution B = 3.5 lakhs As per the gratuity act 1972, gratuity received from by an employee from more than one employer in the same previous year the maximum amount of gratuity exempt from tax u/s 10(10)(iii) cannot exceed Rs 350000/7. How is past service defined under EPS 1995? Solution: The past service under the EPS scheme means the period of service by a member before the eps became effective i.e. up to 15 11 -1995 as a member of the erstwhile Family Pension Scheme.

8. Net Worth of a client is simply the excess of assets over liabilities. 9. Defined benefit pension is determined on a percentage of pre retirement salary 10. Rate of wage to be paid to a monthly rated employee, under payment of gratuity act is.____ for each completed year of service or part thereof in excess of six months. a. b. c. d. Solution: A 15 days wages 30 days wages 7 days wages 26 days wages

11. Rate of wage to be paid to a seasonal employee, under payment of gratuity act is a. b. c. d. Solution: C 12. A scheme providing pension benefits as per IT provision is called ____ a. b. c. d. Solution B 13. Mr. Vishal is working with A ltd from October 1st 1993. He is entitled to a basic salary of Rs. 6000 per month. DA is 40% of basic salary. He retired on Jan 1st 2007. Benefits received Gratuity Rs. 98000, Pension from Jan 1st 07 Rs. 2000pm. Payment from recognized PF Rs. 300000. Encashment of earned leave for 150 days Rs.36000. he was entitled to 40 days leave for every completed year of service. Got 50% of his pension commuted in lumpsum w.e.f. April 1st 07 and receives Rs. 120000/- as commuted pension. Vishal contributes Rs.900 per month to RPF to which his employer contributes an equal amount. What will be the amount of uncommuted pension for Vishal that will form part of his total income for Ay07 08? Pension scheme Superannuation Retirement Scheme Retirement income scheme 15 days wages 30 days wages 7 days wages 26 days wages

a. b. c. d.

4000 Nil 1000 6000

Solution D = 6000 Pension received for Jan, Feb., & March 14. Mr. Manish is working with A ltd from October 1st 1993. He is entitled to a basic salary of Rs. 6000per month. DA is 40% of basic salary. He retired on Jan 1st 2007. Benefits received Gratuity Rs. 98000, Pension from Jan 1st 07 Rs. 2000pm. Payment from recognized PF Rs. 300000. Encashment of earned leave for 150 days Rs.36000. he was entitled to 40 days leave for every completed year of service. Got 50% of his pension commuted in lumpsum w.e.f. March 1st 07. What amount of leave encashment is eligible for tax exemption for Manish? a. Actual encashed b. Nil c. 10 months average salary d. None of the above Solution = D Least of 10 x 8400 = 84000 Actual received = 36000 Statutory limit = 300000 13 x 40 = 520days, actually taken 150 days, 520 150 = 370, As per act 13 x 30 = 390, therefore balance 390 370 = 20, 8400 x 20/30 = 5600. 15. Mohan is an employee having an average balance of Rs.90000 in the PF for the last 12 months. Unfortunately he dies in an accident. What will be the maximum amount of insurance cover payable to his nominee under the EDLI scheme? a. 60000 b. 90000 c. 48750 d. 41250 Solution C 35000 + 25% (90000 35000) = 48750 16. If nominal rate of interest is 12% and compounding is done monthly what would be the effective rate of return? a. 12.58% b. 12.61% c. 12.68% d. 12.75% Solution C N = 12, I = 12%, effective rate = 12.682%

17. Given a) Personal consumption of goods & services = 150 Billion, B) Government expenditure = 70 billion C) Private sector fixed capital expenditure = 50 billion D) Export receipts = 75 billion E) Import expenditure = 80 billion. What will be the value of GDP? a. 425 billion b. 270 Billion c. 295 billion d. 265 billion Solution : D 150+70+50+75-80 = 265 billion 18. Kiran decides to accumulate 50 lakhs when he retires. He is 30 years old and wants to retire at 55. Interest rate is 9% pa. & inflation is 5%pa.Compounding to be done on annuity certain basis. If Kiran could save only Rs.45000 pa for the first 10 years how much does he need to save for the next 15 years to meet his retirement nest egg? a. Rs. 85477pa. b. Rs. 79250 pa. c. Rs. 98450 Pa. d. Rs. 147008pa. Solution A Mode = end, n = 10, i = 9%, pmt = -45000, FV = 683681.8373, PV = -683681.8373, mode = end, n = 16, I = 9%, FV = 5000000, pmt = 85477 19. Mr. Laxman, NRI working in USA for 5years. He is aged 40, and saving Rs. 8lakhs pa. For the past 5 years and hopes to save for the next 10 years. He would like to return to India 10 years from now. The inflation adjusted monthly income requirement for Laxman is Rs. 80000 in the year in which he returns to India. Established that inflation is 3% for the next 30 years. Life expectancy is 70 years. If the estimated spend is Rs. 90000pm. For the family & inflation is 4% how long will his savings last? Investments earn 6% pa., compounding done on annuity certain basis? a. No, lasts until 81 years old b. last until 71 years old, c. will exhaust before 71 Solution B Mode = end, n = 15, pmt = -800000, I = 6%, FV = 18620775.91; Mode = end, I = ((1.06/1.04)-1)/12, = 0.160256410, PV = -18620775.91, pmt = 90000, n =251.56 months; n =20.96 years. i.e. 50 + 20.96 = 70.96 years 20. Mr. Swapnil will start receiving a pension of Rs.12000 pa., exactly 10 years from now. He will receive this pension for 20 years every year end. If interest rate is 12% what is the present worth of his pension? a. Rs 32323

b. Rs.28860 c. Rs.83667 d. Rs.32257 Solution B Mode = end, I = 12%, n = 20, pmt = 12000, PV = 89633, Mode = end, I =12%, n = 10, FV = 89633, pv= 28859.5 21. Bond Face value Rs 1000 and coupon rate 8% with market rate is 10%. If the bond is perpetual the value of the bond is ______. a. 800 b. 750 c. 1000 d. 1200 Solution A Price = 80/10% = 800 22. Ram has received gratuity of Rs. 75000/- in the year 2000. In the year 2006 he received an additional gratuity of Rs. 300000/-. The amount exempt from tax will be _____ 1. 350000 2. 275000 3. 300000 4. None of the above Solution B, 350000 75000 = 275000 If an employee who has received gratuity in any earlier year from his former employer(s), receives gratuity from another employer in a later year, the afore said limit of Rs.350000/- will be reduced by the amount(s) of gratuity exempted from tax under sector 10(10)(iii) in any earlier year(s) 23. An employee contributes Rs.5200/- each month with a similar amount contributed by the employer to a recognized provident fund. What is the tax benefit available to the employee on the contributions made by the employer? a. Amount in excess of 12% is included in gross salary. b. Exempt upto 12% of salary. c. Not exempt but also not taxable every year. d. No tax benefits available 1. All of the above 2. only C & D 3. Only A & B Solution: 2 24. An individual invests Rs 50000/- in his PPF account. While having a deduction of Rs. 24000/- during the year as provident fund. Further insurance policies of Rs.

50000/- premium were purchased. What is the tax benefit available at the end of the year? Solution Rs.100000/25. An investor buys a pension policy from an insurance company after paying a premium of Rs. 24000/- Other insurance premium paid is to the extent of Rs 94000/-. What is the total amount of benefit of tax available u/s 80c & 80 ccc? a. 118000 b. 100000 c. 94000 d. nil Solution B Rs 100000/The maximum limit of benefits available under section 80C & 80CCC is Rs.100000. 26. A person has a basic salary of Rs. 35000/- per month & there is a provident fund deduction of 10% towards a recognized provident fund. How much will be the tax benefit available for the individual? a. Deduction u/s 80C available on Employees contribution from gross total income subject to certain limits b. Exemption upto 12% of salary. Amount in excess of 12% is included in gross salary. c. Interest on Provident fund exempt u/s 10 upto 9.5% p.a. Interest credited in excess of 9.5% p.a. is included in gross salary. d. No exemption available. 1. A B C 2. A B C D 3. A B 4. A C Solution 1 ( A B C)

27. An employee wants to decide between contributing a sum of Rs 3500/- pm either to the provident fund or the public provident fund. The tax benefit in the two cases for the employee will be _____ a. Exempt under section 80CCC b. Exempt under section 80D c. Exempt under section 80C d. No tax benefits available Solution C Exempt under section 80C

28. An employee joined service in May1999 & left it on Jan2006. at the time of resignation he received a sum of Rs.254000/- as accumulated balance from recognized provident fund. The amount that is taxable is ____. a. Rs 254000 b. NIL, The entire amount is exempt c. None of the above Solution B As he has completed more than 5 years of continuous service in the company the entire amount received is exempt.. The accumulated balance due & becoming payable to an employee participating in a recognized PF shall be exempt in the following cases 1. If the employee has rendered continuous service with his employer for a period of 5 years or more ,or 2. If, though he has not rendered such continuous service of 5 years, the service has been terminated a) by reason of such employees ill health or b) by the contraction or discontinuance of the employers business or c) or other cause beyond the control of the employee, or 3. If, on the cessation of his employment, the employee obtains employment with any other employer, to the extent the accumulated balance due and becoming payable to him is transferred to his individual account in any recognized fund maintained by such other employer. 29. Jatin is 40 yrs old. He earns Rs 2.8lakhs a year currently. If his earnings rise 8% for the next 15 years & he wants a replacement ratio of 80%, what will be his requirement at the time of retirement? a. 701555 b. 710566 c. 710655 d. 701655 Solution B Mode = end, n = 15, I = 8%, PV = -280000, FV = 888207.3520, 80% x 888207.3520 = 710566 30. The current income of a person is Rs. 4lakhs pa. He wants a sum of Rs. 5lakhs at the replacement ratio of 80% in 5 years time. By what rate should his income increase to provide such a benefit? a. 7.33% b. 8.33% c. 9.33% d. 10.33% Solution C Fv =500000/80% = 625000 Mode = end, n = 5, PV = -400000, FV = 625000, I = 9.33%

31. If a person wants a sum of Rs 42000/- pm at a replacement ratio of 70%, what should his normal earning be at the time of retirement?____ a. 4.2 lakh b. 5.6 lakhs c. 6.4 lakhs d. 7.2lakhs Solution D 42000 x 12/ 70% = 7.2 lakhs 32. A sum of Rs. 4.8lakhs at 60% replacement will mean a person has to earn before retirement a sum of ______ a. 5lakhs b. 6lakhs c. 7lakhs d. 8lakhs Solution D 480000/60% = 800000 33. The pension received by a person is Rs. 6000/- pm. If the other income after deductions is Rs. 1lakh for a male individual, then the pension received is taxable to the extent of ____ a. 6000 b. 12000 c. 72000 d. 44000 Solution C 6000 x 12 = 72000 34. If the income from a senior citizen savings scheme for a 63 year individual is Rs105000/- and there is no other income then the amount of ------ is taxable. a. Nil b. 5000 c. 10000 d. 500 Solution A 35. If there is a deposit of Rs. 70000/- in a 8% pa deposit for 270 days then will there be a tax deduction at source on the interest paid? a. No b. Yes c. Cant say d. None of the above Solution A

36. A person wants to invest Rs 4 lakhs in a post office monthly scheme.

a. b. c. d. e.

it can be done in a single name it can be done in a joint name there is no bonus on maturity both a & b are correct only B is correct

solution D As per the POMIS rules up to 4.5 lakhs can be invested in a single name & up to 9 lakhs can be invested in a joint name 37. A provident fund account is closed and the amount is withdrawn by the individual after 4 years. The amount received here is ______ a. taxable b. not taxable c. none of the above solution A 38. A provident fund account is transferred after 4 years of operation to another employer the earning in this case is ____ a. taxable b. not taxable c. none of the above solution B 39. An amount of Rs.100000/- is invested in a notified pension scheme of a mutual fund --- will be allowed as a deduction under section 80c a. 100000 b. 10000 c. Contributions to pension funds of mutual funds do not qualify for 80C deduction. d. Nil Solution A 40. Neha received inheritance of Rs 2.5 lakhs, wants to withdraw equal amounts at the beginning of each month for next 7 years at 10%pa compounded monthly. What amount she should start withdrawing per month? a. 4116 b. 2266 c. 4150 d. 2626 Solution A Mode beginning, n = 7x12, i=10%/12, PV = -250000 PMT = 4116

41. Ram has bought the share of ABC ltd. For Rs. 80000. They grow to Rs. 90000 in 182days. a. What is the return?

b. What is the annualized return? 1. 12.5%, 25.06% 2. 11.5%, 25.06% 3. 12.5%, 25% 4. None of the above Solution A 10000/80000 = 12.5%, 12.5x 365/182 =25.06% 42. Ms. Madhu is 40 yrs old to retire at 65. Life expectancy is 75 yrs. She will require 5000 in 1st month after retirement. Inflation 4% p.a. , rate of return 7% . What is the corpus required to meet the expenses after retirement. Will the corpus be enough to fund her retirement if she saves up to Rs. 30,000 pa (at the end of the year) a. 1478597, Yes b. 1687498, No c. 1984571, No d. 1562027 , Y Solution D Retirement corpus pmt = 15000, n =10 x 12, I = ((1.07/1.04)-1)/12 = .24038, PV = 1562031 Savings corpus mode end, n = 25, I =7%, pmt = -30000, FV = 1897471.131 43. Nirav wants to retire at 45 and he wants to maintain his present standard of living. He spends 325000 a year. He is expected to live up to 85. Inflation 4% expected return 7% pa . How can he achieve this? He is at present 30 yr. What is the nest egg required at age 45 and what amount shall he save every year to meet this plan? His present investment is Rs.10, 00,000. a. Nest egg and saving required will be 25100065 and 541093 resp. b. Nest egg and saving required will be 12773065 and 438300 resp. c. Nest egg and saving required will be 13785155 and 438781 resp. d. Nest egg and saving required will be 14773065 and 740530 resp. Solution C Find FV45, n = 15, I = 4%, PV = -325000, FV = 585306.64 = pmt for 40 years, I= 2.88461545, n =40, pmt = 585306.64, PV 45 = 13785170 Savings per year I = 7%, n =15, PV = -1000000, FV = 13785170, pmt = 438781.06 44. Mira aged 30 saves Rs. 15000 per year (at the end) in Bank FD earning 8.25% P.a. compounded annually until she retires at 58. Life expectancy is 80 yrs. Calculate the Corpus on the date of retirement? What is the fixed amount she can Withdraw at the beginning of each yr until 80 in case she wishes to exhaust her corpus completely. Inflation rate is 5% pa. a. 1424894, 89458 b.1348974, 87498 c. 1491655, 137767 d. 1491655, 91613

Solution D Mode = end, pmt = -15000, i= 8.25%, n =28, FV = 1491655, Mode = beginning, n =22, I = 3.09%, PV = -1491655, pmt = 91612.558 45. Rekha 20 yrs retired at 45. Life expectancy is 70. She requires 55000 in the first month of her retirement. Inflation rate is 4% pa; rate of return is 6% pa. What will be the saving per year required in order to meet this. a. 232584 b. 235789 c. 236478 d. 238615 Solution D Mode = end, n = 25 x 12, pmt = 55000, I = 0.019230769 / 12, PV =- 13091516.01 N= 6%, n = 25, FV = 13091516.01, pmt = 238615.37

46. Mr. Prakash is 35 yrs old wants to retire at 60. Life expectancy is 80 yrs. He will require 25000 in 1st month after retirement. Inflation is 5% p.a , rate of return 8% . What is the corpus required to meet the expenses after retirement. Will the corpus be enough to fund her retirement if he saves upto Rs. 48,000 pa (at the end of the year) a. 4129874, Y b. 4784962, N c. 4566431, N d. 4378154, Y Solution C Mode = end, n = 20 x 12, i = 2.857142857 / 12, pmt = 25000, PV = -4566431 Saving corpus; mode = end, n = 25, I = 8%, Pmt = -48000, FV = 3509085.118 47. Kalpesh wants to accumulate Rs. 50 Lakh when he retires. He is 30 yrs and wants to retire at 55 yrs. Interest rate 9 % pa, inflation 5% pa.. Compounding to be done on annuity Certain basis. After 10 yrs of his saving 45000pa. Kalpesh now realizes he can now earn12 % pa on fresh investment and also maintain a saving amount of 72000 pa for the rest of the working life. Will he be able to accumulate the amount required? What will his corpus be? a. Y , 51.74 Lakh b. N, 48.75 Lakh c. Y, 52.5 Lakh d. N, 49.50 Lakh Solution A Mode = end, n = 10, I =9%, pmt = -45000, FV = 683681.8373; PV = - 683681.8373, n =15, I = 9%, mode = end, FV = 2490299.1 Mode = end, I = 12%, n = 15, pmt = -72000, FV = 2684139.456 Total savings 2490299.1 + 2684139.456 = 5174438.556

48. Krishna aged 30 saves Rs. 10000 per year (at the end) in Bank FD earning 8% p.a.comp annually until he retires at 60. Life expectancy is 80 yrs. Corpus on the date of retirement? What is the fixed amount he can withdraw at the beginning of each yr until 80. Presuming he wishes to leave his heirs an estate of Rs.200,000 .Inflation rate is constant at 6% pa. a. 1105694, 61375 b. 1132832, 12787 c. 1054875, 108423 d. 1132832, 59040 Solution D Mode = end, n = 30, I =8%, pmt = -10000, FV = 1132832. Mode = beginning, n =20, FV = 200000, I = 1.88%,PV = -1132832, Pmt = 59040 49. Manoj 30 yrs employee earning salary of Rs. 300000. He started saving 10 % of his salary, at the end of the year, in a saving plan which yields 6% interest pa. His salary increases by 5% pa. If Manoj intends to prepone his retirement to the age of 55 yrs and needs to have the same amount of accumulated saving as at the age of 60 yrs. What percentage of his salary should he start saving to achieve his goal. a. 14.04% b. 3.56% c. 15.70% d. 6.70% Solution C = 30000 x [(1.06)^30 (1.05)^30] / (0.06 -0.05) = 4264646.395 4264646.395 = x [(1.06)^25 (1.05)^25]/ 0.06 0.05 =47096.32336 47096.32336 / 300000 = 15.698% 50. Ramesh retired from PTC completed service of 29 yrs 9 month. His salary at retirement was Rs. 10,500 p.month. While average salary of preceeding 10 months worked out to Rs. 9800 p.m. The actual amount of gratuity received by him at retirement was Rs.325000. Calculate the amount of gratuity exempt from tax assuming he is covered by provision of Payment of gratuity Act 1972. a. 294000 b. 181730 c. 315000 d. 175673 Solution B Covered under the gratuity act therefore gratuity exempt will be least of actual received, max limit Rs.300000, or calculated as per the act as 15 X 10500 X 30 = 181730 26 51. Shinde invested Rs. 72000 at the rate of interest of 5%. After 7 yrs the ROI is 5% compounded half yearly. After 3 yrs ROI is 6% compounded quarterly. What will he get after 15 years. a. 140000 b. 155000 c. 148251

d. 158242 Solution D Mode = end, PV = -72000, n = 7, I =5%, FV = 101311.2304, PV = -101311.2304, n = 3 X 2, I = 5% / 2, FV = 117489.9671, PV = -117489.9671, n = 5 x 4, I = 6% / 4, FV = 158241.9504 52.Mukesh retires from service received Rs 10 Lakh in total tax free retirement benefit. He will receive Rs. 10,000 p.m. as pension but his proposed expenditure are 200% of .What he receives. He is a senior citizen and wants to invest his lumpsum amount in a scheme which provides safety of principle, regular and max. interest. He is open to invest in MF (MIP ) or like fund but only as second option. Assuming return on debt and MF (MIP) be around 8% pa. What is the maximum amount he should put in Post Office Senior citizen Saving Scheme so that his tax liability remains NIL without requiring to invest under section 80C. If he invests rest of his fund in MF(MIP) by what % he needs to reduce his proposed monthly expenditure to meet the deficit in interest earning. a. 833000, 13.65% b. 1000000, 13.65 c. 1000000, 10% d. 833000, 13.18% Solution: D Expenses = 20000pm i.e. 240000 pa. Income = 10000pm i.e. 120000 pa Shortfall = 10000pm i.e. 120000 pa Nil tax bracket for senior citizens current slab = 195000 To achieve NIL tax liability he needs to invest 195000 120000 = 75000 Amount to be invested in Senior Citizen Scheme = 75000/0.09 = 833333; approx = 833000 Amount to be invested in MIP = 1000000 833000 = 167000 Income from Pension = 120000 Interest from Senior Citizen scheme = 833000 x 9% = 74970 Interest from MIP =167000 x 8% = 13360 208330 Reduction in expenses = 240000 208330 = 13.18% 240000

53. Harinder has heard that changes in inflation rate might have a significant impact on his real saving. Currently he would buy the car of his dreams for Rs. 3 Lakh. He wants to estimate the amount he may need to buy the car in 8 yrs time. The inflation rate for the period are expected to be 5% for 1st four year, , 4 % for next four year value of the car is expected to fall by 10 % every time over a period of 4 yrs. If he buys the car in 8 yrs then what would be the same real amount that he would need to have saved. a. 345539

b. 400000 c. 383932 d. 426591 Solution A Mode = end, PV = -300000, n = 4, I = 5%, Fv = 364651.8750 PV = -328186.6875, n = 4, I = 4%, FV = 383932, Amount needed as value of car will reduce by 10% = 345539 54. Ram aged 35 saves Rs. 30000 per year (at the end) in Bank FD earning 7% p.a. comp annually until he retires at 60. Life expectancy is 70 yrs. Corpus on the date of retirement ? What is the fixed amount he can withdraw at the beginning of each yr until 70. Presuming he wishes to leave his heirs an estate of Rs. 150,000 . Inflation rate is constant at 5% pa. a. 1897471, 242337 b. 2147854, 214598 c. 1745897, 192712 d. 1945875, 199145 Solution A Mode = end, n = 25, I = 7%, FV = 1897471, As inflation is constant we will use nominal rate of 7% in post retirement phase. PV = -1897471, FV = 150000, N=10, I =7%, PMT = 242337 55. An employee retires on 10th Mar 06. Salary during 1- May-2005 to 31-Dec-2005 is Rs. 26000 pm. 1- Jan-2006 to 28-Feb-2006 is Rs. 26500 pm. Besides he receives Rs. 400/- pm. as DA. 20 % Basic Pay as DA (not part of salary for computing of retirement benefit). He is entitled to 6% commission on sales achieved by him (during 1-May-2005 and 28-Feb-2006 turnover achieved by the employee is 2577860) . He received a gratuity of Rs.750000/- and has worked for 26 yrs and 5 month in the organization calculate the tax free and taxable amount of gratuity resp. a. 350000, 741142 b. 350000, 400000 c. 750000, 341142 d. 750000, 400000 Solution = B x 26 x 41967 = 545573, As payment is least of 350000, actual received or as calculated Least is 350000, is exempt gratuity therefore balance 400000 is taxable 56. Mohan invests Rs 10000 in mutual fund on 1/1/2000.He receives cash dividend of Rs 200,300,200,300 on31/12/2000, 2001, 2002 & 31/12/2003.He sold the fund for Rs 15000/- on 1/1/2004.What is IRR? a. 12.81%, b. 10.21% c. 6.83%, d. 8.53%

Solution A Cash flow -10000, 200,300, 200,18000 IRR =12.81% 57. Amit aged 30 yrs has an estimated present expense of Rs 360,000p.a.If inflation is 5% in first ten years, 6% in next ten years, & 6.5% for rest of the years. What would be household expenses required at age 60 yrs if to maintain standard of leaving is 80% post retirement. a. 11.5 lacs, b. 15.77lacs, c. 16.31lacs, d. 8.53lacs Solution B Mode = end, n = 10, I =5%, PV = -360000, FV= 586402.0656 N =10, I = 6%, PV = -586402.0656, FV= 1050156.788 N= 10, I =6.5%, PV = -1050156.788, FV = 1971288.652 Retirement expenses = 80% x 1971288.652 = 15.77 lakhs 58. Mr. Kalpesh decides to accumulate Rs 50 lacs when he retires. He is at present 30 yrs old & wants to retire at 55yrs .Assume interest rate 9% and inflation 5% compounding to be done on annuity certain basis. If Mr. Kalpesh could save only Rs 45000/- for the first ten years ,how much he need to save next 15 years to meet his retirement corpus. a) 147008 p.a, b) 85477 p.a, c) 98450p.a, d) 79250 p.a Solution B N=10, mode = end, I = 9%, pmt = -45000, FV = 683681.8373 N = 15, mode = end, I = 9%, PV =-683681.8373, FV = 5000000, Pmt = 85477 59. Mr. Alpesh decides to accumulate Rs 50 lacs when he retires. He is at present 30 yrs old & wants to retire at 55yrs .Assume interest rate 9% and inflation 5% compounding to be done on annuity certain basis. If Mr. Kalpesh could save only Rs 45000/- for the first ten years , After this he realizes that he can now earn 12% p.a on fresh investment and also increases his investments amount to 72000/p.a.for the rest of the working life. Will he be able to save the required amount? What will be his corpus at retirement? a. Yes, 52.50 lacs, b. No, 48.75 lacs, c. no, 49.50 lacs, d. yes, 51.74 lacs Solution D Mode = end, n = 10, I =9%, pmt = -45000, FV = 683681.8373 PV = -683681.8373, n= 15, mode = end, I =9%, FV = 2490299 Mode = end, I = 12%, n = 15, pmt = -72000, Fv = 2684139 5174438.55

60. U have been appointed as financial planner of Mr. Ranjit. He is 35 yrs old at present and would be retiring at age 60 yrs.He expects to live up to 80 yrs.You have to plan in such a way that Ranjit starts getting 25000 pm from the very day he retires and keep receiving till his survival. If an expected return during accumulation is 7%p.a, what is saving required per year to meet his retirement needs. a) 47646, b) 50982, c) 47924, d) 51280 Solution D Mode begin, n = 20 x 12, I = 7%/12, pmt = 25000, pv = -3243372.611 Mode = end, n = 25, I = 7%, FV = 3243372.611, pmt = 51279.39 61. Ms Shalini purchases a money back policy at age of 30 yrs. She gets survival benefit of Rs 1 lac each in 5,10,15th yr of policy and Rs 4,00,000 as maturity benefit at the end of 20 yrs. She invests entire benefits of this policy at 8%p.a.What is accumulated value of this investments to her at age 60 yr. a) 3037260, b) 1511247, c) 2331730, d) 1531730 Solution C Pv = -100000, n = 25, I = 8%, FV1 = 684848 PV = -100000, n = 20, I = 8%, FV2 =466096 PV = -100000, n = 15, i = 8%, FV3 = 317217 PV = -400000, n = 10, i = 8%, FV 4 = 863570 Total 2331730 62. Mr. Sumit present age 25 yrs wants to retire at 55 yrs .Life expectancy is 70 yrs.He estimates that he will require 35000 pm after retirement. Inflation is 4.5% and return on investment is 6.5%.What will be saving required per annum? a) 86987, b) 63358, c) 82689, d) 85479 Solution B Mode = end, n = 15 x 12, I = 0.159489633%, pmt = 35000, PV = -5472593 FV =5472593, mode = end, n = 30, I = 6.5%, pmt = 63358 63. Mr Branden presently 30 yrs old, wants to retire at 45 yrs.He wants to maintain present standard of leaving .He is currently spends around Rs 3,25,000 p.a.He expects to survive upto 85 yrs .Inflation is 4% and expected return on investments are 7%. What will be nest egg required at age 45 yrs? What amount, he needs to save every year in the end to meet his plan of retirement. His present savings are

around 10 lacs. a). 25100065, 541093 b) 12773065, 438300 c) 13785155, 438781 d) 14773065, 740530 Solution C Mode = end, I =4%, n=15,PV = -325000, FV = 585307 Mode end, I = 2.88%, n =40, pmt = 585307, PV = -13785155 Mode = end, I = 7%, n = 15, PV = -1000000, FV = 13785155, pmt = 438781 64. Mr. Laxman worked for 25 yrs and gets retirement money of 15 lacs as Retirement benefit. He is also entitled to get pension of 5000pm.His expenses are Rs 1,00,000 p.a. He wants to keep 3 lac aside for emergency requirement as contingent fund and invest remaining amount between senior citizen saving schemes & Mutual fund MIP in such a manner that he is able to save Rs 20,000/- under sec 80C out of the income generated from the senior citizen scheme. Assume 8% return on MF MIP, which is a 2nd option of investment. What is the amt of investment in MF-MIP & Sr.Citizen scheme? a) 533,000& 967000 b) 667000 & 533000 c) 778000 & 422000 d) 533000 & 667000 Solution C Retirement corpus = 1500000 300000 = 1200000. Annual pension received =5000 x 12 = 60000 Min slab = 110000 80c = 20000 130000 Income = 60000 Shortfall to earn =70000 Investment in senior citizen scheme = 70000/ .09 = 778000 As he wants to invest balance in mutual fund i.e. 1200000 778000 = 422000 in mutual fund.

65. Monthly pension of an employee under EPS95, if pension able service is 30 yrs & pension able salary is Rs6000/a) Rs6000 b) Rs2571 c) 3000 d) 1500 Solution B Monthly pension = pensionable salary x pensionable service/ 70 = 6000x30/70 = 2571

66. Mr. Sharma is retiring at 60. He has got a nest egg of Rs 20 lacs. Inflation is around 6.5 % & interest rate is 10%. Calculate rate of growth. a) 3% b) 3.29% c) 3.35% d) 2.5% Solution B Real rate = 1 + nominal rate = 3.29% 1 + inflation 67. Security A 12% return & 7% SD, Security B 18% return & 8% SD. Which one would the client prefer? a. Sec B b. Sec A Solution A 68. Aman 25 yrs old, working with a Pvt. Ltd Co. since 2004. He receives Rs 20,000/- towards leave encashment in 2007 which is equal to 1 mths salary. What is the taxable component? a) 4000 b) 12500 c) 20000 d) Nil Solution C Leave encashment during service is fully taxable. 69. Anup receives Rs 62,000/- as gratuity. He retires on 31.1.2007 after completing service of 29 yrs 8 mths. The Co. is covered under Gratuity Act 1972. His salary at the time of retirement is 3100/- p.m, what is the amt of gratuity exempt from tax? a) 53653 b) 8346 c) 62000 d) Nil Solution B 30 x 3100 x 15/26 = 8346 70. As a person approaches the retirement date almost all his contributions to the retirement plan should be kept in investments which offer him the _____ a) Highest safety of capital B) Mostly into equity related avenues c) In banks to provide liquidity d) None of the above Solution A State whether true or false

71. A senior citizen savings scheme cannot be extended for next three years after maturity of account. a. The above statement is True b. The above statement is False c. Neither of the above Solution B Senior citizen savings scheme can be extended for 3 years after maturity of account 72. Senior Citizen Savings scheme the interest is payable monthly. a. The above statement is True b. The above statement is False c. False interest is payable Quarterly d. Neither of the above Solution C 73. The post office recurring deposit account is normally for _____ years. a. 3 b. 6 c. 5 d. 2 Solution C 74. Can the post office recurring account be continued on maturity? a. No b. YES c. Yes for another 5 years on year to year basis d. Yes for three blocks of 5 years each. Solution C 75. Which post office savings scheme earns rebates for 6 to 12 months advance Deposits? a. Post office Time deposit account b. Post office savings account. c. Post office recurring deposit scheme d. There is no such a scheme Solution C 76. Monthly pension of an employee under EPS95, if pension able service is 35 yrs & pension able salary is Rs6000/Ans a) Rs6000 b) Rs2571 c) 3000 d) 1500 Solution C Monthly pension = pensionable salary x pensionable service/ 70 = 6000x35/70 = 3000

77.The current yield of a bond, with coupon rate of 7.5% & market price of Rs.105

is ___ a. 7.143% b. 7.413% c. 6.143% d. Nil Solution A 7.5 / 105 = 0.0714285 X 100 = 7.143 88. The current yield of a bond with coupon rate of 11.5% is 10.773%. How much is the price of this bond. a.. 100 b. 106.75 c. 107.76 d. Nil Solution B 11.5 / 10.773 = 1.06748 X 100 = 106.75 79. Calculate the bonds price with face value of 100 & coupon rate of 11.5% maturing in 3 years, if the ytm is 9%. a. 106 b. 106.3208 c. 100 d. 105 Solution B Fv = 100, I = 9%, n = 3, pmt = 11.5, PV = 106.3208 80. What is the price of a bond whose face value is 100 coupon is 10% maturing in 3 years if the ytm is 14% a. 90.717 b. 90 c. 91 d. 100 Solution A N = 3, I = 14%, FV = 100, pmt = 10 ,PV = 90.717 81. Given the following details about a mutual fund calculate the net asset e. investments 350 crores f. other assets 40 crores g. accrued expenses 12 crores h. accrued income 8.75 crores i. current liabilities 3.45 crores j. other liabilities 9.6 crore, Solution 373.7 crores (350+40+8.75-12-3.45-9.6)

82. A mutual fund incurs Rs.8.5 crores as initial issue expenses for mobilizing Rs. 60 crores. Calculate the amount that can be amortized each year and for how many years can the initial issue expenses be amortized? k. l. m. n. 72 lakhs, 5 years 75 lakhs, as long as the fund exists 72 lakhs, 3 years only None of the above

Solution A (Only 6% can be charged to the fund as initial expenses so 6%* 60 crores = 3.6 crores, this amt can be amortized over 5 years, i.e. 3.6/5 = 72 lakhs) 83. A scheme incurs 15 crores as initial issue expenses for mobilizing 200 crores for a closed end scheme with a tenor of 6 years. How will this expense be treated in the books of the scheme? o. 386415 p. 386451 q. 384615.38 r. Nil Solution C 6% can be charged as expenses i.e. 6%*200 = 12 cr its a close ended scheme therefore 12 cr has to be amortized on a weekly basis for the tenor of the scheme each week the amortization would be 12/ (6*52) = 384615.38 84. A mutual fund holds a 91 day treasury bill, issued at 94.5 redeeming at 100. if there are 26 days to maturity what is the value of the instruments on its books? s. 98.25 t. 98.43 u. 94.5 v. 100 Solution B The value of all instruments with less than 182 days remaining to maturity has to be done on accrual basis the difference between issue price & redemption 100-94.5 = 5.5, accrual per day = 5.5/91, number of days since issuance 91- 26 value on accrual basis when 26 days remain to maturity = 94.5 +[(5.5/91)*(91-26)] = 98.43 85. A mutual fund holds a 91 day treasury bill, issued at 95.25 redeeming at 100. if there are 34 days to maturity what is the value of the instruments on its books? a. 98.22 b. 98 c. 95.25 d.100 Solution A 95.25 + [(4.75/91)*(91 34)] = 98.22

86.A mutual fund holds a 182 day treasury bill, issued at 89.75 redeeming at 100. if there are 180 days to maturity what is the value of the instruments on its books? a. 89.86 b. 90 c. 100 d. 88 Solution A 89.75 + [(10.25/182)*(182 180)] = 89.86

87.A mutual fund holds a 90 day commercial paper, issued at 92.72 redeeming at 100. if there are 45 days to maturity what is the value of the instruments on its books? a. 96.63 b. 96.36 c. 93.63 d. 93.36 Solution B 92.72 + [(7.28/90)*(90 45)] = 96.63

88. NAV of a fund on march312001 was 14.45. 3 months later the NAV had grown to 15.25. using the percentage change in NAV method find out the annualized return a .22.145 b .22 c .21.145 d .nil Solution A (15.25 -14.45)/14.45 *100 = 5.536% *12/3 89. NAV of a fund on Jan 2001 was 15.865. 6 months later the NAV had grown to 16.35. using the percentage change in NAV method find out the annualized return a. 6.114 b. 6 c. 5.114 d. 5 Solution A (16.35 -15.865)/15.865 *100 = 3.057% *12/6 90.Mahesh bought units of a mutual fund for 18.225 at the end of the year the worth of his holding was 18.725 & he had received a dividend of 12 % compute his return using simple total return method a.9.238%

b. 9.328% c. 9 d. Nil Solution B Dividend received 10 * 12% = 1.20, ((18.725-18.225) + 1.20)/18.225 =.9.328 91.Mahesh bought units of a mutual fund for 23.75 he receives a dividend of 12.5% which he reinvests in the fund at the then prevailing NAV of 23.5 at the end of the year the worth of his holding was 24.65 compute his return using reinvestment method a. 9.3102% b. 9.13% c. 9% d. Nil Solution A Dividend received = Rs.1.25 , extra unit purchased = 1.25/ 23.5 = 0.05319, total units = 1.05319, value of holding at the end of the year = 1.05319 x 24.65 = 25.96117, HPR = (25.96117 23.75 )/ 23.75 = 9.3102% 92. Mukesh retired from 30th June 2007. He received pension of 6000pm till 31st Dec 2007, since 1st Jan 08 he gets 60% of pension commuted for 90000. Calculate amount of pension includable in salary income for AY 08 09 Assume no gratuity received. . 52800 . 58200 . 58000 . Nil Solution B Uncommuted pension from June to dec07 = 6000 X 6 =36000 Total pension = 90000/60% = 150000, can commute 50% as gratuity not received i.e. 75000 as exempt amount, but commutes 90000 therefore 90000 75000 = 15000 = taxable.; reduced pension from Jan to March = 2400 x 3 = 7200, total pension 36000 + 15000 + 7200 = 58200

93. Amar is 30 years old wants to retire at 45 years. He deposits 25000pa at the year end for 15 years, earning an interest rate of 5%. He wants 75000 pa post retirement. How long will his corpus suffice him? . 8 years . 9 years . 8.59years . 7 years Solution C Mode = end, n = 15, pmt = -25000, FV = 539464.09

Mode = begining, I = 5%, pmt = 75000, PV = -539464.09, N =8.59 years 94. Seema aged 35 years requires 20lakhs at age 55. If she starts today using an annual stepped up method. Calculate how much she needs to save in first year to ensure that her investment plan corresponds with salary growth. Assume interest rate 8%, growth 10% and inflation 4%. a. 17596 b. 17922 c. 17900 d. 18900 Solution B FV = 2000000, n = 20, I = 8%, PV = -429096.4148 N=20, rate = (8-10)/1.10 = -1.8181, PV = -429096.4148, pmt = 17922 95. What is the present value of an investment that will be worth Rs 10000 in 10 years based on an inflation rate of 4% pa. a. 3855 b. 5584 c. 6756 d. 8203 Solution C FV = 10000, n = 10, I = 4%, PV =6756 96. Ajay opens a saving account on 15th April 2007 with a deposit of Rs.2000, there after he deposits Rs.80 into the account fortnightly. How much will the account have on 15th April 08. Interest rate of 8.3% a. 13933 b. 14933 c. 15933 d. 16933 Solution C PV = -2000, r = 8.3%/26, n = 5 x 26, FV = 3026.7402 Pmt = -80, r = 8.3%/26, n = 5 x 26, FV = 12906.2474 Total accumulation = 3026.7402 + 12906.2474 = 15932.9876 97. Jagan, a 45 years old executive earns a salary of Rs. 68000 pa. He will retire at the age of 55 years & his life expectancy is 76 years. He expects his current salary to grow by 6% pa. The inflation rate is expected to be 4% pa. throughout his lifetime & the expected return on investment is 5% pa. during retirement. If Jagans target replacement ratio is 70% of his last drawn salary. Calculate how much he needs as his retirement fund. ( Assume salary increments are given at the beginning of each year.) a. 1613980 b. 1629499 c. 197321 d. 1992183

Solution B FV of salary = 68000 x (1.06^10) = 121777.6434 Expected expenses = 70% x 121777.6434 = 85244.35 Capital requirement for retirement = pmt = 85244.35, I = (5%-4%) / 1.04 = 0.9615, n =21, PV = 1629499.2989 98. To avail the withdrawal from the PF for the purchase of a ready built house / flat or construction of a house the member should have completed 5 years of membership of the fund and the purchase should be in favour of a. Member or member & spouse with minimum balance in members account should be Rs.1000/b. Member only with minimum balance in members account should be Rs.1000/c. Spouse only with minimum balance in members account should be Rs.1000/d. Member & spouse with minimum balance in members account should be Rs.1000/Solution A 99._____ is a single premium market linked deferred pension plan from ICICI Prudential Life Insurance Company. a. Forever life b. Life link Pension c. Life time Pension d. New Jeevan Dhara I Solution B 100. Profit Sharing plans are ______ a. Defined Benefit Plans b. Defined Contributions Plans c. Hybrid Plans d. None of the above. Solution B 101. Profit sharing plans are a. Generally not structured to provide a monthly pension benefit at retirement b. Its aimed at offering the employees a way of participation in the companys profit 0 Both statement are false 0 Both statements are True 0 Only A is true 0 Only B is true Solution 2 102. The administrative charges for the EPS are paid by

a. b. c. d. Solution C

The employer only The employer & the government The Central Government only No administrative charges are paid.

103. Definition of "Family" for the Employees Pension Fund means (i) wife in the case of male member of the Employees' Pension Fund; (ii) husband in the case of a female member of the Employees' Pension Fund ; and (iii) sons and daughters of a member of the Employees' Pension Fund ; (iv)The expression "sons" and "daughters" shall include children 2[legally adopted by the member]. a. None of the above b. All of the above c. Only 1 & 2 d. Only 1,2,& 3 Solution B 104. Administrative expenses for Employees Provident Fund scheme payable by the employer every month will be ______ a. 1.10% of total wages on which Provident fund is recovered b. a minimum of Rs 5/- payable by the employer every month. . . . . Solution III Only A Only B Both A & B None of the above

105. Arun has taken a loan of Rs.450000/-, with a down payment of 20% and installments to be paid for 15 years at the rate of 12% per annum. Calculate the estimated monthly installment. a. 4300 b. 4320 c. 3240 d. 4000 Solution = B Find PMT PV = (450000 90000 ) = 360000, n = 15 x12, i = 12/12 = 1%, pmt = 4320.605

You might also like